Autor Nachricht
franz
BeitragVerfasst am: 05. Apr 2009 09:03    Titel: Näherung möglich?

Hallo!

Vielleicht kommt man mit r << L zu einer näherungsweisen Lösung alpha =ca arccos (g / L*omega^2) ?

F.
Gast20180108
BeitragVerfasst am: 01. Apr 2009 15:20    Titel:

so heute hat einer erzählt, dass der prof. letztes jahr gesagt hat, dass es mathematisch so nicht lösbar ist! finde ich natürlich besonders gut, dass man das auch einmal erfährt! also müsste deine lösung stimmen as_string. wollte noch mal nachfragen ob man hier die statik als spezialfall der dynamik annimt wegen der gleichförmigen bewegung also v=const??
sax
BeitragVerfasst am: 31. März 2009 21:24    Titel:

Sorry, hstte nicht bedacht das noch r0 dazukommt. Ich sehe im Moment auch keine analytische Lösung.
Gast20180108
BeitragVerfasst am: 31. März 2009 20:48    Titel:

ah komisch gedacht sorry! aber lustig ein nachmittag mit diesem bsp.*gg*
as_string
BeitragVerfasst am: 31. März 2009 20:20    Titel:

Hä? Wenn alpha gegeben wäre, dann müsste man es ja nicht mehr ausrechnen.

Wie gesagt: theoretisch kannst Du das schon ausrechnen, wenn Du einfach sagst, dass ist. Da hast Du nur noch alpha als Unbekannte. Numerisch kein Problem. Analytisch weiß ich nicht so genau...

Gruß
Marco

PS: Man könnte natürlich die Kleinwinkel-Näherung nehmen und einfach sagen: . Aber ob das im Sinne des Erfinders ist?
Gast20180108
BeitragVerfasst am: 31. März 2009 20:15    Titel:

meine vermutung war ja auch schon, dass in der angabe was vergessn wurde! eben alpha! denn dann müsste man ihn in der abhängigkeit von w leicht berechnen können wenn ich mich net täusche??
as_string
BeitragVerfasst am: 31. März 2009 20:10    Titel:

Hallo!

Ja, eben. Im Prinzip hast Du dann eine Gleichung mit einer Unbekannten alpha. Das Problem ist jetzt nur, dass es einmal der Sinus ist und einmal der Tangens. Diese transzendente Gleichung kann man mE nicht elementar lösen. Zumindest fällt mir da gerade kein Trick ein... Numerisch geht natürlich immer.

Gruß
Marco
Gast20180108
BeitragVerfasst am: 31. März 2009 19:46    Titel:

ja mitn tangens wenn ich mich nicht irre! aber woher kenne ich die zentrifugalkraft? aus m*r*w^2??? das ist wieder ein wenig komisch da der abstand r hier ich nenne das r in der zeichnung r0. r=r0+L*Sin(alpha) und da habe ich das alpha auch wieder drinnen??
as_string
BeitragVerfasst am: 31. März 2009 19:37    Titel:

sax hat Folgendes geschrieben:
Du kennst Gewichtskraft und Zentrifugalkraft,

Naja, die kennte er eigentlich ja noch nicht, weil der Abstand von der Rotationsachse wieder von alpha abhängt und damit auch die Zentrifugalkraft. Mathematisch wirds glaube ich dann etwas aufwändiger.

Gruß
Marco
sax
BeitragVerfasst am: 31. März 2009 19:33    Titel:

Du kennst Gewichtskraft und Zentrifugalkraft, die Seilkraft hat natürlich die Richtung des Seils.
Die Skizze sollte dann wie unten aussehen. Kannst du erkennen wo dort der Winkel alpha auftauchen muss? Kannst du alpha dann aus Fz und Fg errechnen?
Gast20180108
BeitragVerfasst am: 31. März 2009 19:18    Titel:

ja hätte ich schon versucht aber ich kenne ja alpha nicht.
Gast20180108
BeitragVerfasst am: 31. März 2009 19:07    Titel:

ich studiere maschinenbau 2.sem. wieso?
sax
BeitragVerfasst am: 31. März 2009 19:05    Titel:

Polorkoordinaten brauchst du hier nicht.
Du musst das Kräftegleichgewicht zwischen Gewichtskraft, Zentrifugalkraft und Seilkraft aufstellen. Am besten wäre du machst dir erst mal eine Skizze, in der du die drei Kräfte einzeichnest.
planck1858
BeitragVerfasst am: 31. März 2009 18:53    Titel:

Für welche Klassenstufe ist denn diese Aufgabe?
Gast20180108
BeitragVerfasst am: 31. März 2009 18:46    Titel: Karussell (Dynamik)

Hallo!

Habe heute wieder einmal ein Problem mit dynamik! und zwar ein Karussell!

Gegeben: Am Umfang einer Kreisscheibe (Radius r) sind Seile (Länge L) mit Endmassen m befestigt. Durch Drehung der Kreisscheibe schwenken die Seile nach außen. Die Seile sind als masselos anzusehen.

Gesucht: Seilkraft S und Ausschlagswinkel alpha als Funktion der Winkelgeschwindigkeit w.

Könnte mir hier jemand einen Denkanstoß geben ich weiß nicht wie ich hier rechnen soll! würden sich hier polarkoordinaten anbieten????

Powered by phpBB © 2001, 2005 phpBB Group